Deriving General Lorentz Boost Equation

In summary: Lorentz boost matrix in the x-direction. To get a general Lorentz boost in an arbitrary direction, we can use the rotation matrix R = \begin{bmatrix}c_yc_z & -c_ys_z & s_y & 0\\ c_x s_z+c_z s_x s_y & c_x c_z -s_x s_y s_z & -c_y s_x & 0\\ -s_x s_z+c_x c_z s_y & c_z s_x+c_x s_y s_z & c_x
  • #1
HJ Farnsworth
128
1
Greetings,

I have been having trouble deriving the equation for the general Lorentz boost for velocity in an arbitrary direction. It seems to me that given the 1D Lorentz transformations...

matrix for Lorentz transformation in x-direction, X:
{{1/sqrt(1-v^2), -v/sqrt(1-v^2), 0, 0},
{-v/sqrt(1-v^2), 1/sqrt(1-v^2), 0, 0},
{0, 0, 1, 0},
{0, 0, 0, 1}}

matrix for Lorentz transformation in y-direction, Y:
{{1/sqrt(1-v^2), 0, -v/sqrt(1-v^2), 0},
{0, 1, 0, 0},
{-v/sqrt(1-v^2), 0, 1/sqrt(1-v^2), 0},
{0, 0, 0, 1}}

matrix for Lorentz transformation in z-direction, Z:
{{1/sqrt(1-v^2), 0, 0, -v/sqrt(1-v^2)},
{0, 1, 0, 0},
{0,0,1,0},
{-v/sqrt(1-v^2), 0, 0, 1/sqrt(1-v^2)}}

...all I should have to do is replace each v in X with vx, each v in Y with vy, and each v in Z with vz, and then multiply the matrices to get ZYX as the general Lorentz boost equation. This should sequentially compute the Lorentz transformation in the x-direction with the x-velocity, then the y-direction with the y-velocity, then the z-direction with the z-velocity. However, when I do that, the answer I get only vaguely resembles the general Lorentz boost equation found on Wikipedia (http://en.wikipedia.org/wiki/Lorentz_transformation#Matrix_form).

I can't find the derivation for the general boost matrix anywhere, and more importantly, I have no idea why the simple math I used is wrong - it seems conceptually correct to me.

Please tell me what the flaw in my logic is, and how to correct it.

Thanks for any help you can give.

-HJ Farnsworth
 
Physics news on Phys.org
  • #2
By the way, in case this wasn't clear in my previous post, the vx, vy, and vz in the previous post were for 3-velocity vector v = vxex + vyey + vzez, where the e's are orthonormal basis vectors in the x-, y-, and z-directions.
 
  • #3
HJ Farnsworth said:
Please tell me what the flaw in my logic is, and how to correct it.

In general, the product of two non-colinear boosts is not a boost. To find a boost in an arbitrary direction from a boost [itex]B[/itex] in the x direction, compute
[tex]R B R^{-1},[/tex]
where [itex]R[/itex] is the rotation matrix that rotates the x direction into the arbitrary direction.
 
  • #4
Offhand, I'd say that what you should do to get the general boost is to perform some rotation R, so that the rotated coorinates xr, yr, zr have the boost going along one of the axes - say the x-axis for specificity.

Then you do the boost.

Then you perform the inverse rotation R^(-1) such that R(-1)R = a unit matrix, and you should have the general boost along an arbitary direction.

It's important to get the right order, and I'm not totally sure I've gotten that right in my text description. Basically, I expect it to go like this:

[tex]
R^{-1} \Lambda R
[/tex]

where R is the rotation matrix. The composition process works from right to left, first you apply R, then do the boost [itex]\Lambda[/itex], then you do the inverse of R, see http://en.wikipedia.org/w/index.php?title=Transformation_matrix&oldid=431791983

"Composing and inverting transformations"There's probably an easier way.

[add] I see George snuck an answer in ahead of me, they seem to be equivalent except that the matrix R is defined diffrently.
 
  • #5
Hey,

Thanks for the quick responses . Your answers make perfect sense - in fact I can't believe I didn't think of that. Ah well, we all have off days I guess.

I still have one question though. I understand why RBR-1 works - it rotates the coordinate axes to be in the easiest direction, does the standard Lorentz transformation, then rotates the coordinate axes back so we have the transformed coordinates in our original coordinate system. However, I am still not quite clear on why a series of orthogonal boosts does not work. How come the product of two non-colinear boosts is not a boost? Please be explicit/detailed in telling me why my original thought process in my first post was wrong.

Again, thanks very much.

-HJ Farnsworth
 
  • #6
More precisely, the product of two non-parallel boosts is not purely a boost. Generically, the result will be a boost and a rotation. For example, if you boost in the X direction and then boost in the Y direction, the result will be a boost in some direction in the XY plane, combined with an additional rotation about the Z axis.

This can be seen from the Lorentz algebra:

[tex]\begin{gather*}[J_a, J_b] = \varepsilon_{abc} J_c \\ [K_a, K_b] = -\varepsilon_{abc} J_c \\ [J_a, K_b] = \varepsilon_{abc} K_c \end{gather*}[/tex]

where [itex]J_a[/itex] are the rotations, and [itex]K_a[/itex] are the boosts. In particular, two boosts do not generically commute (as per second line).
 
  • #7
HJ Farnsworth said:
Hey,

Thanks for the quick responses . Your answers make perfect sense - in fact I can't believe I didn't think of that. Ah well, we all have off days I guess.

I still have one question though. I understand why RBR-1 works - it rotates the coordinate axes to be in the easiest direction, does the standard Lorentz transformation, then rotates the coordinate axes back so we have the transformed coordinates in our original coordinate system. However, I am still not quite clear on why a series of orthogonal boosts does not work. How come the product of two non-colinear boosts is not a boost? Please be explicit/detailed in telling me why my original thought process in my first post was wrong.

Again, thanks very much.

-HJ Farnsworth
Sorry for digging this thread up, but why does RBR-1 work?

Is B the vx boost matrix? If so, why does it cause it go in the 'easiest direction' when it acts on the rotation matrix. I can see why you then have to do the inverse to go back to the original coordinate basis, but why is the resulting matrix now in a more general form? As you can guess, my understanding of matrices is pretty shaky.
EDIT: Using that method I can't seem to get the right result anyway... Is the rotation matrix supposed to be: {{coshx, -sinhx, 0, 0}, {-sinhx, coshx, 0, 0}, {0, 0, 1, 0}, {0, 0, 0, 1}} ?
 
Last edited:
  • #8
Given a 3D velocity v = (vx, vy, vz) with v = |v|, [tex]
\textbf{B} = \begin{bmatrix}
\gamma & -\gamma v & 0 & 0\\
-\gamma v & \gamma & 0 & 0\\
0 & 0 & 1 & 0\\
0 & 0 & 0 & 1
\end{bmatrix}
[/tex]
Choose two unit vectors f and g so that v, f and g are all orthogonal to each other.
[tex]
\textbf{R} = \begin{bmatrix}
1 & 0 & 0 & 0\\
0 & v_x/v & f_x & g_x\\
0 & v_y/v & f_y & g_y\\
0 & v_z/v & f_z & g_z
\end{bmatrix}
[/tex]

Now work out RBRT
 
  • #9
DrGreg said:
Now work out RBRT
Thanks for the help. I did what you said but without understanding why it works.

Comparing it to the required result, I get something very similar but which requires equations such as the following to be true:

vx2/v2 + fx2 + gx2 = 1

and

-(vxvy)/v2 = fxfy + gxgy
Why are these true? (And why does the method even work at all?)
 
Last edited:
  • #10
3nTr0pY said:
Thanks for the help. I did what you said but without understanding why it works.

Comparing it to the required result, I get something very similar but which requires equations such as the following to be true:

vx2/v2 + fx2 + gx2 = 1

and

-(vxvy)/v2 = fxfy + gxgy


Why are these true?
The way I defined R ensures that its columns are orthonormal. It is a fact about square matrices, maybe not immediately obvious but nevertheless true, that orthonormal columns imply orthonormal rows. Or, to put it another way, [itex]\mathbf{R}^T\mathbf{R} = \mathbf{I}[/itex] implies [itex]\mathbf{RR}^T = \mathbf{I}[/itex]. Your equations above follow from this.
3nTr0pY said:
(And why does the method even work at all?)
The matrix R was chosen as an orthogonal matrix satisfying[tex]
\textbf{R}^T \, \begin{bmatrix} 0 \\ v_x \\ v_y \\ v_z \end{bmatrix}
= \begin{bmatrix} 0 \\ v \\ 0\\ 0 \end{bmatrix}
[/tex]or equivalently
[tex]
\textbf{R} \, \begin{bmatrix} 0 \\ v \\ 0\\ 0 \end{bmatrix} = \begin{bmatrix} 0 \\ v_x \\ v_y \\ v_z \end{bmatrix}
[/tex](and, of course, leaving the time dimension unmodified).
 
Last edited:
  • #11
Great. Thanks again for your help.
 

1. What is the General Lorentz Boost Equation?

The General Lorentz Boost Equation is a mathematical formula that describes the transformation of space and time coordinates between two reference frames that are moving relative to each other at a constant velocity. It is a fundamental concept in special relativity and is used to calculate the effects of time dilation and length contraction.

2. How is the General Lorentz Boost Equation derived?

The General Lorentz Boost Equation is derived from the principles of special relativity, which state that the laws of physics are the same for all observers in uniform motion. It involves applying the Lorentz transformations to the coordinates of an event in one reference frame to calculate its coordinates in another frame.

3. What are the variables in the General Lorentz Boost Equation?

The variables in the General Lorentz Boost Equation are the relative velocity between the two reference frames (v), the time in the first frame (t), the length in the first frame (x), and the speed of light (c). These variables are used to calculate the time (t') and length (x') in the second frame.

4. What are the applications of the General Lorentz Boost Equation?

The General Lorentz Boost Equation has many applications in modern physics, including explaining the effects of time dilation and length contraction in special relativity. It is also used in particle physics, astrophysics, and other fields to calculate the behavior of objects moving at high speeds.

5. Are there any limitations to the General Lorentz Boost Equation?

While the General Lorentz Boost Equation is a powerful tool in understanding the effects of special relativity, it does have some limitations. It only applies to objects moving at constant velocities and does not account for acceleration or gravitational effects. It also does not apply to objects moving at speeds close to the speed of light, where more complex equations must be used.

Similar threads

  • Special and General Relativity
Replies
6
Views
1K
  • Special and General Relativity
Replies
22
Views
1K
  • Special and General Relativity
Replies
4
Views
1K
  • Special and General Relativity
Replies
3
Views
984
  • Special and General Relativity
Replies
10
Views
600
  • Special and General Relativity
Replies
1
Views
1K
  • Special and General Relativity
Replies
10
Views
1K
  • Special and General Relativity
Replies
14
Views
1K
  • Special and General Relativity
3
Replies
101
Views
3K
  • Special and General Relativity
Replies
8
Views
792
Back
Top